PTB.S1.Q16 - In a town containing a tourist attraction, hotel and attraction (I picked c )

IlikepizzaIlikepizza Alum Member
edited May 2023 in Logical Reasoning 5 karma

PTB S1 Q16

Comments

  • JesseWeNeedToCookJesseWeNeedToCook Alum Member
    edited May 2023 137 karma

    Looking for the answer choice that doesn't weaken the argument:

    A) incorrect: More tourist attractions implies people travelling to the town aka increased amounts of money being spent at the hotels and restaurants but not necessarily at this attraction

    B ) Incorrect: Offers an alternative explanation. Same amount of people buying passes, however utilizing hotels and restaurants more.

    C) Incorrect: Offers an alternative explanation. Hotel and restaurant profits have increased more rapidly because they started charging more, meanwhile the attraction has not started charging more.

    D) Correct: definitely doesn't weaken the argument, looks like it is borderline strengthening (blocks out an alternative explanation). If people's amount of time staying at hotels hasn't changed, then why are hotels seeing increased profits while the attraction is not seeing this same rate of increase.

    E) Incorrect: Denies the conclusion by stating the passes have ID's which are checked.

Sign In or Register to comment.